www.vorkurse.de
Ein Projekt von vorhilfe.de
Die Online-Kurse der Vorhilfe

E-Learning leicht gemacht.
Hallo Gast!einloggen | registrieren ]
Startseite · Mitglieder · Teams · Forum · Wissen · Kurse · Impressum
Forenbaum
^ Forenbaum
Status Mathe-Vorkurse
  Status Organisatorisches
  Status Schule
    Status Wiederholung Algebra
    Status Einführung Analysis
    Status Einführung Analytisc
    Status VK 21: Mathematik 6.
    Status VK 37: Kurvendiskussionen
    Status VK Abivorbereitungen
  Status Universität
    Status Lerngruppe LinAlg
    Status VK 13 Analysis I FH
    Status Algebra 2006
    Status VK 22: Algebra 2007
    Status GruMiHH 06
    Status VK 58: Algebra 1
    Status VK 59: Lineare Algebra
    Status VK 60: Analysis
    Status Wahrscheinlichkeitst

Gezeigt werden alle Foren bis zur Tiefe 2

Navigation
 Startseite...
 Neuerdings beta neu
 Forum...
 vorwissen...
 vorkurse...
 Werkzeuge...
 Nachhilfevermittlung beta...
 Online-Spiele beta
 Suchen
 Verein...
 Impressum
Das Projekt
Server und Internetanbindung werden durch Spenden finanziert.
Organisiert wird das Projekt von unserem Koordinatorenteam.
Hunderte Mitglieder helfen ehrenamtlich in unseren moderierten Foren.
Anbieter der Seite ist der gemeinnützige Verein "Vorhilfe.de e.V.".
Partnerseiten
Weitere Fächer:

Open Source FunktionenplotterFunkyPlot: Kostenloser und quelloffener Funktionenplotter für Linux und andere Betriebssysteme
Forum "Funktionalanalysis" - Operatortheorie
Operatortheorie < Funktionalanalysis < Analysis < Hochschule < Mathe < Vorhilfe
Ansicht: [ geschachtelt ] | ^ Forum "Funktionalanalysis"  | ^^ Alle Foren  | ^ Forenbaum  | Materialien

Operatortheorie: Frage (beantwortet)
Status: (Frage) beantwortet Status 
Datum: 18:41 Di 20.07.2010
Autor: Sierra

Aufgabe
Zu berechnen ist das Spektrum des Operators T: [mm] l^{2}->l^{2} [/mm] mit
[mm] T:(a_{1},a_{2},a_{3},...) \mapsto ((1-1)a_{1}, (1-\bruch{1}{2})a_{2}, (1-\bruch{1}{3})a_{3}, [/mm] ... , [mm] (1-\bruch{1}{n})a_{n}) [/mm]
Außerdem ist noch zu zeigen, dass T nicht kompakt ist

Hallo,

soweit ich weiß ist das Spektrum eines Operators die Menge aller Eigenwerte des Operators.
Nun weiß ich allerdings gar nicht, wie man diese hier bestimmen kann.. ? Kann man den Operator irgendwie in Matrixform bringen und dann eine Eigenwertgleichung lösen?

Nun sind Operatoren mit endlich dimensionalem Bild sind kompakt, würde das bedeuten, dass es unendlich viele Eigenwerte gibt, da der Operator, laut Aufgabenstellung, nicht kompakt ist?

Da ich ziemlich ahnungslos in diesem Gebiet bin, bin ich über jede Hilfe dankbar...

Gruß
Sierra

        
Bezug
Operatortheorie: Antwort
Status: (Antwort) fertig Status 
Datum: 23:18 Di 20.07.2010
Autor: meili

Hallo Sierra,

> Zu berechnen ist das Spektrum des Operators T: [mm]l^{2}->l^{2}[/mm]
> mit
>  [mm]T:(a_{1},a_{2},a_{3},...) \mapsto ((1-1)a_{1}, (1-\bruch{1}{2})a_{2}, (1-\bruch{1}{3})a_{3},[/mm]
> ... , [mm](1-\bruch{1}{n})a_{n})[/mm]
>  Außerdem ist noch zu zeigen, dass T nicht kompakt ist
>  Hallo,
>  
> soweit ich weiß ist das Spektrum eines Operators die Menge
> aller Eigenwerte des Operators.

Das Spektrum eines Operators kann noch mehr sein als die Menge aller Eigenwerte des Operators.

> Nun weiß ich allerdings gar nicht, wie man diese hier
> bestimmen kann.. ? Kann man den Operator irgendwie in
> Matrixform bringen und dann eine Eigenwertgleichung
> lösen?

Zum besseren Verständnis des Operators T kann man eine Matrix aufstellen. Die Zeilen und Spalten dieser Matrix sind Folgen.
Zur Bestimmung des Spektrums des Operators T, ist der Operator
$T - [mm] \lambda \\ [/mm] id$ zu untersuchen. Siehe auch []Spektrum eines Operators

>  
> Nun sind Operatoren mit endlich dimensionalem Bild sind
> kompakt, würde das bedeuten, dass es unendlich viele
> Eigenwerte gibt, da der Operator, laut Aufgabenstellung,
> nicht kompakt ist?
>  
> Da ich ziemlich ahnungslos in diesem Gebiet bin, bin ich
> über jede Hilfe dankbar...
>  
> Gruß
>  Sierra

Gruß meili

Bezug
        
Bezug
Operatortheorie: Antwort
Status: (Antwort) fertig Status 
Datum: 09:13 Mi 21.07.2010
Autor: fred97

Ergänzend zu Meili:

Seien [mm] e_1,e_2,e_3, [/mm] ... die Einhetsvektoren in [mm] $l^2$ (e_j [/mm] hat ander j-ten Stelle eine 1 und sonst nur Nullen).

Die Folge [mm] (e_k) [/mm] ist beschränkt. Zeige: die Folge [mm] (Te_k) [/mm] enthält keine konvergente Teilfolge.

Damit ist T nicht kompakt


FRED

Bezug
Ansicht: [ geschachtelt ] | ^ Forum "Funktionalanalysis"  | ^^ Alle Foren  | ^ Forenbaum  | Materialien


^ Seitenanfang ^
www.vorkurse.de
[ Startseite | Mitglieder | Teams | Forum | Wissen | Kurse | Impressum ]